You are on page 1of 63

VOL4

Dear Aspirants

Team iLearn IAS is proud to present our weekly current affairs magazine.
Keeping in tune with the dynamic demands of the civil service examination, we
bring to aspirants on-going topics that are of relevance, particularly for the
mains exam.

The objective of this magazine is to make the aspirants exam ready in an


exhaustive list of topics. To this end, we provide detailed background notes on
the topics followed by an in-depth analysis of the issues, covering aspects such
as institutional mechanisms, policies, policy gaps, areas of focus, reforms
needed, suggestions and way forward.

We have also included all relevant Prelims topics from the past week.

In order to help the aspirants in monitoring their preparation, we have


included probable question that can be expected for the mains and Prelims
examination. You may also visit our integrated current affairs platform
www.ilearncana.com for your current affairs preparation.

We wish you all the very best and welcome you on board in our endeavors.

Happy Learning!
Team iLearn
TABLE OF CONTENTS
1. STRATEGIC PETROLEUM RESERVES ............................................................................... 4
1.1 WHY IN NEWS? ................................................................................................................ 4
1.2 SYLLABUS: ........................................................................................................................ 4
1.3 PRACTICE QUESTION:....................................................................................................... 4
1.4 ANALYSIS: ......................................................................................................................... 4
1.5. iTips: ................................................................................................................................ 6
2. SOLID WASTE MANAGEMENT IN INDIA ......................................................................... 7
2.1 WHY IN NEWS? ................................................................................................................ 7
2.2 SYLLABUS:......................................................................................................................... 7
2.3 PRACTICE QUESTION: ....................................................................................................... 7
2.4 ANALYSIS: ......................................................................................................................... 7
2.5. iTips: .............................................................................................................................. 12
3. SHUKRAYAAN MISSION.............................................................................................. 13
3.1 WHY IN NEWS? .............................................................................................................. 13
3.2 SYLLABUS: ...................................................................................................................... 13
3.3 PRACTICE QUESTION:..................................................................................................... 13
3.4 ANALYSIS: ....................................................................................................................... 13
3.5. iTips: .............................................................................................................................. 15
4. METHANOL ECONOMY PROGRAM .............................................................................. 16
4.1 WHY IN NEWS? .............................................................................................................. 16
4.2 SYLLABUS: ...................................................................................................................... 16
4.3 PRACTICE QUESTION:..................................................................................................... 16
4.4 ANALYSIS: ....................................................................................................................... 16
4.5. iTips: .............................................................................................................................. 21
5. DEEP OCEAN MISSION ................................................................................................ 22
5.1 WHY IN NEWS? .............................................................................................................. 22
5.2 SYLLABUS: ...................................................................................................................... 22
5.3 PRACTICE QUESTION:..................................................................................................... 22
5.4 ANALYSIS: ....................................................................................................................... 22
5.5. iTips: .............................................................................................................................. 25
WEEKLY PRELIMS COMPILATION..................................................................................... 26
• Pushkarulu ................................................................................................................... 27
• Inner line Permit .......................................................................................................... 27
• Deemed Forests ........................................................................................................... 28
• Brahmaputra River ....................................................................................................... 30
• Barikot Ghundai ........................................................................................................... 31

2 | iLearn IAS
• Roshni Act .................................................................................................................... 32
• Swiss Challenge method .............................................................................................. 34
• Pangda Village and Doklam ......................................................................................... 35
• Unified Mobile Application for New-age Governance (UMANG) ................................ 36
• Sentinel-6 satellite ....................................................................................................... 37
• Doddagaddavalli hoysala Temple ................................................................................ 38
• Lachit Borphukan ......................................................................................................... 40
• Chang’e-5 ..................................................................................................................... 41
• Guardian Drones .......................................................................................................... 42
• Pradhan Mantri Kisan Sampada Yojana (PMKSY) ........................................................ 43
• Nagaland Tamarillos .................................................................................................... 45
• SDG Investor Map by Invest India ................................................................................ 46
• Antimicrobial resistance (AMR) ................................................................................... 47
• National Command, Control, Communication and Intelligence Centre (NC3I) ........... 49
• Sangai deer................................................................................................................... 50
• Dooars .......................................................................................................................... 51
• Blue Tides ..................................................................................................................... 52
• Brain Electrical Oscillation Signature Profiling (BEOSP) ............................................... 53
• Farmers Producer Organisations ................................................................................. 54
• Climate Change Knowledge Portal .............................................................................. 56
• Neom City..................................................................................................................... 57
• Cosmos Malabaricus .................................................................................................... 58
• Natural Gas sector in India........................................................................................... 59

3 | iLearn IAS
1. STRATEGIC PETROLEUM RESERVES
1.1 WHY IN NEWS?
• India has invited global firms to invest in its strategic petroleum reserves (SPRs).
1.2 SYLLABUS:
GS 3: Infrastructure: Energy
1.3 PRACTICE QUESTION:
Q. Identify the major Strategic petroleum reserves of India. Discuss the significance of
Strategic petroleum reserves in India’s energy security?
1.4 ANALYSIS:
STRATEGIC PETROLEUM RESERVES (SPR):
• SPR are crude oil inventories (or stockpiles), usually held by the government of a
particular country, to safeguard the economy and national security during an energy
crisis.
• The concept was first mooted in 1973 in the US, after the 1973 oil crisis. Later crises,
like the 1979 oil crisis and Iraqi invasion of Kuwait, gave further impetus to the idea.
MAJOR OIL CRISES:
• The 1973 oil crisis, also called the "first oil shock", began when the members of the
Organization of Arab Petroleum Exporting Countries proclaimed an oil embargo,
targeted at nations perceived as supporting Israel during the Yom Kippur War. The
targeted nations were the United States, Canada, Japan, the Netherlands, the United
Kingdom and South Africa.
• The 1979 Oil Crisis, also known as the Second Oil Shock, was an energy crisis caused
by a drop in oil production in the wake of the Iranian Revolution.
• First Gulf War: In August 1990, Iraq invaded Kuwait, sending the price of oil soaring
from about $34 per barrel to nearly $77. After a U.S.-led military coalition succeeded
in removing Saddam Hussein's Iraqi forces from Kuwait in early 1991, the price fell to
about $37.
• The 2008 Oil Shock: A series of events that cut global production sharply led to a
significant spike in oil prices. Eg: Venezuela cut off sales to Exxon Mobil in a legal
battle over nationalization of that company's properties, labor strikes reduced
production in Nigeria and the U.K.'s North Sea oil fields and Mexico endured a severe
decline in production from one of its major oil fields. As a result, the price of oil rose
to above $165 by mid-2008.
• U.S. Shale Oil Revolution: U.S. oil and gas output increased by about 57% over the
past decade until early 2020 as advances in fracking technology. Partly as a result,
the price of crude oil fell from about $87 per barrel in early 2010 to just under $51 by
January 2020.
INDIA AND SPRs:
• India started development on a strategic crude oil reserve in 2003.
• Today, the Government has set up 5.33 million metric tons (MMT) of strategic crude
oil storages, enough to provide 10 days of consumption.

4 | iLearn IAS
• They are at three locations namely, Visakhapatnam, Mangalore and Padur (near
Udupi). In the second phase, two more such caverns will be set up: at Chandikhole
(Odisha) and Bikaner (Rajasthan). It will add 11.57 days more.
• The storages are constructed in underground rock caverns.
o Crude oil to these caverns is supplied by oil exporting giants through long
term agreements. Eg: Half of Mangalore’s storage has been hired by Abu
Dhabi National Oil Co (ADNOC) to store its crude oil.
o Crude oil from these caverns can be supplied to the Indian Refineries either
through pipelines or through a combination of pipelines and coastal
movement.
• Indian Strategic Petroleum Reserves Limited (ISPRL), a Special Purpose Vehicle and
a wholly owned subsidiary of Oil Industry Development Board (OIDB) under the
Ministry of Petroleum & Natural Gas undertakes the construction of the Strategic
Crude Oil Storage facilities.
• Engineers India Limited (EIL) worked as the Project Management Consultant for the
three locations.
NEED OF SPRs:
• Import dependency: Currently India meets 82 % of petroleum demand through
imports. The IEA predicts that by 2020, India could well be the largest oil importer,
increasing the vulnerability to threats of physical supply disruptions.
• Volatile oil markets: The oil market is susceptible to an array of global events, like
tensions in middle east, stock market fluctuations, OPEC policies and USA’s
sanctions. All of this can increase the oil prices drastically, which can severely affect
India’s exchequer.
• Rising demand: Despite the thrust on renewables and natural gas, petroleum
continues to be the primary fuel for India. The World Energy Outlook 2019 published
by International Energy Agency shows that India’s oil demand in total primary energy
is projected to increase from 233 MT in 2018 to 305 MT in 2025.
• Reassure its growth trajectory: All major powers have a strong strategic reserve to
secure their needs. China has around 80 days of oil in storage, while Japan has a
capacity of above 150 days. International Energy Agency (IEA) members maintain
emergency oil reserves equivalent to at least 90 days of net imports.
BENEFITS OF SPRs:
• Energy security: The reserves provide a 22-day supply and the Indian refiners
maintain 65 days of crude storage. Thus, India has an overall reserve oil storage of
87 days, which would serve as a cushion during any supply disruptions.
• Monetary benefit: By stocking up oil at times of price drop and relying on themse
reserves at times of price hike, India can save thousands of crores of rupees. This
strategy is being implemented now, since there is fall in the international crude oil
prices due to COVID crisis and India is filling its strategic reserves at low cost.

5 | iLearn IAS
•Investment avenue: Government has developed plans for commercialisation of the
reserves. For eg: To attract private investment in its SPRs, India recently allowed Abu
Dhabi National Oil Co (ADNOC) to re-export some of its oil stored in Mangalore SPR,
mirroring a model adopted by South Korea and Japan.
• Strategic cooperation: Oil stockpiling and development opens avenues for strategic
collaborations with other nations. For eg: India and the US have signed an MoU on
operation and maintenance of strategic petroleum reserves and prospect the
possibility of India storing oil in the US.
• Safety: Underground rock caverns are considered as the safest means of storing
hydrocarbons. Unlike large storage tanks, the caverns can be easily protected from
threats like fires and sabotages.
CONCLUSION:
• The challenge now is to fill all the caverns. Nearly half of the 5.33 awaits to be filled.
However, the drastic price drops in both the WTI and Brent benchmarks have
presented India with a unique opportunity to stock up on its crude oil reserves.
• Considering the volatility of the oil prices and the increasing geopolitical challenges
in the Indian Ocean region, with a pandemic on the loose and the inevitable
economic challenges added to the mix, it is of critical national importance that India
completes the Phase 2 of SPRs with as little pilferage of time and resources as
possible.

1.5. iTips:
• For Mains, the content can also be used in questions related to energy security,
vulnerability of India’s energy sector and
• For Prelims, questions on the locations of various reserves, their features and
countries involved in their development can be expected.

6 | iLearn IAS
2. SOLID WASTE MANAGEMENT IN INDIA
2.1 WHY IN NEWS?
Prime Minister of India launched a campaign in August 2020 to free India of garbage
2.2 SYLLABUS:
GS 3: Conservation, Environmental Pollution and Degradation, Environmental Impact
Assessment.
2.3 PRACTICE QUESTION:
Q. “The key to efficient solid waste management is to ensure community participation”.
Analyse.
2.4 ANALYSIS:
CURRENT SCENARIO:
• Waste management rules in India are based on the principles of "sustainable
development", "precaution" and "polluter pays"
• According to World Bank report India is world’s highest waste generating nation
with 62 million tonnes generated annually
• The per capita waste generation in Indian cities ranges from 200 grams to 600
grams per day, which is one of the highest in the world
• Only about 80% of the municipal waste gets collected and only 22-28 % of this
waste is processed and treated. Rest 80% is dumbed in landfills

CHALLENGES:
• High waste generation:
o With rapid urbanisation, rising population, high dependence on plastics >>
waste generation exceeds in sustainable treatment
• Presence huge informal sector in the waste collection:
o Difficult to implement the rules
• Ineffective waste collection:
o Absence of waste segregation at source into biodegradable and non-
biodegradable
• Unscientific disposal methods:
o Lack of sophisticated technology for waste disposal and widespread burning
>> GHG emissions and air pollution
• Failures of waste management rules:
o They fail to incentivize and impose a strict penalty in case of poor
implementation.
o The rules have not pushed for decentralized management of waste but have
encouraged centralized treatment such as waste to energy, the present state
of which is not good in the country.
• Poor landfill management:

7 | iLearn IAS
o Landfills generate 20% of methane gas emissions in India and pollute
surrounding land, ground water and air.
• Lack awareness:
o People lack proper awareness regarding the need of better waste disposal for
societies health and well being
• Low participation from civil society:
o Despite immense potential in waste management sector, participation from
non-profits or community is limited.
• Ill-focussed spending:
o Three-fourth of solid waste management budget is allotted to collection
and transportation, leaving leaves very little for processing or resource
recovery and disposal.
• Inefficiency of Local Bodies:
o Lack of funds and inadequate technical expertise, informalization of rag
pickers, absence protective gears for workers >> results in poor waste
management
• Waste to energy plants in India are not operating to their full potential.

IMPACT OF IMPROPER SOLID WASTE DISPOSAL:


• Health Impacts:
o Burning of waste at dumb sites >> release of toxic gases and fine particles >>
respiratory illness
o Disease causing vectors >> breeding ground
o Spread of epidemics through stray animals
• Water pollution:
o Surface water contamination >> through run off from landfills
o Ground water contamination >> through leaching
• Global warming:
o Open landfills releases methane from decomposition. Methane is a major
contributor to global warming
• Inefficient land usage:
o Lack of proper treatment and processing >> increase in demand for
expansion of landfills in many cities ( ex: Mumbai, Delhi) >> many hectares of
land in cities becomes wasteland
GOVERNMENT INITIATIVES:
• SOLID WASTE MANAGEMENT RULES (SWM), 2016:
o Segregation at source:
▪ The rules have mandated the source segregation of waste in order to
channelize the waste to wealth by recovery, reuse and recycle.

8 | iLearn IAS
▪ Waste generators would have to now segregate waste into three
streams- Biodegradables, Dry and Domestic Hazardous waste before
handing it over to the collector.
o Collection and disposal of sanitary waste:
▪ The manufacturers or brand owners of sanitary napkins are
responsible for awareness for proper disposal of such waste
o Collect Back scheme for packaging waste:
▪ Brand owners who sale or market their products in packaging material
which are non‐biodegradable, should put in place a system to collect
back the packaging waste generated due to their production.
o User fees for collection:
▪ The new rules have given power to the local bodies across India to
decide the user fees
o Integration:
▪ Integration of rag pickers, waste pickers and kabadiwalas from the
informal sector to the formal sector would be done by the state
government.
o Zero tolerance policy
▪ The rules also stipulate zero tolerance for throwing; burning, or
burying the solid waste generated on streets, open public spaces
outside the generator’s premises
o Promoting use of compost:
▪ The Department of Fertilizers should provide market development
assistance on city compost
o Promotion of waste to energy:
▪ The rules mandate all industrial units located within 100 km from a
solid waste-based Refuse-Derived Fuel (RDF) plant to make
arrangements to replace at least 5 per cent of their fuel requirement
by RDF so produced.
o Revision of parameters and existing standards:
▪ The landfill site shall be 100 meters away from a river, 500 meters
away from highways etc
o Management of waste in hilly areas
▪ The construction of landfills on hills shall be avoided
o Constitution of a Central Monitoring Committee
▪ The government has also constituted a Central Monitoring Committee
to monitor the overall implementation of the rules.
• CAMPAIGN:
o Swachh Bharat Mission:

9 | iLearn IAS
▪ A country-wide campaign initiated by the Government of India in
2014 to eliminate open defecation and improve solid waste
management.
• TECHNOLOGY:
o CSIR-CMERI has developed a Municipal Solid Waste (MSW) Processing
Facility:
▪ For scientific SWM and to create value added end products
▪ Advanced segregation techniques
• WASTE TO WEALTH:
o GOBAR-Dhan scheme.
▪ Under this scheme Cattle dung, kitchen waste and agricultural waste
can be tapped to create biogas-based energy
▪ The objectives of this initiative is to make villages clean and to
generate wealth and energy from cattle and other waste.
o Government mandates:
▪ Industries to buy electricity from power plants fuelled by solid from
▪ Private fertilizer companies to buy compost that is extracted from
municipal solid waste
• OTHER INITIATIVES:
o Policy on promotion of city compost
o Star rating protocol for Garbage free cities

BEST PRACTICE:
• Local:
o Alappuzha Model – ‘Nirmala Bhavanam Nirmala Nagaram’:
▪ Alappuzha gets recognised by UNEP for its solid waste management
practices
▪ The city has adopted decentralised waste management and is
pushing for 100 per cent segregation in all the 23 wards of the city.
▪ Moreover, as many as 80 per cent households now have biogas
plants and decentralised composting system.
• International:
o South Korea - waste management system
▪ Focused on controlling waste generation and achieving maximum
rates of recycling after 1990s
▪ It has since seen a drastic reduction in MSW generation - from 30.6
million MT in 1990 to 19.3 million MT in 2016.
▪ Meanwhile, landfill and incineration rates have decreased
dramatically from 94% in 1990 to 38% in 2016.
WAY FORWARD:
• Proper segregation of waste at source

10 | iLearn IAS
o The key to efficient waste management is to ensure proper segregation of
waste at source and to ensure that the waste goes through different streams
of recycling and resource recovery.
• Effective use of sanitary landfills:
o Then reduced final residue should be deposited scientifically in sanitary
landfills.
o Sanitary landfills are the ultimate means of disposal for unutilised municipal
solid waste from waste processing facilities and other types of inorganic
waste that cannot be reused or recycled.
• Energy-from-waste:
o Energy-from-waste is a crucial element of SWM because it reduces the
volume of waste from disposal also helps in converting the waste into
renewable energy and organic manure
• Waste-to-compost and bio-methanation plants:
o Biodegradable component of India’s solid waste is currently estimated at a
little over 50 per cent
o Installation of waste-to-compost and bio-methanation plants would reduce
the load of landfill sites
• Concept of common waste treatment facility
o The concept of common waste treatment facility is being widely promoted
and accepted as it uses waste as a resource by either using it as a co-fuel or
co-raw material in manufacturing processes
o For example: Bio-medical waste (management and handling) rules, 1998
prescribe that there should be a Common Biomedical Waste Treatment
Facility (CBWTF) at every 150 kms in the country
• Governance:
o Civic bodies have to redraw long term vision in solid waste management and
rework their strategies as per changing lifestyles
o They should reinvent garbage management in cities so that we can process
waste and not landfill it
o It is reported that almost 80 per cent of the waste at Delhi landfill sites
could be recycled provided civic bodies start allowing ragpickers to segregate
waste at source and recycle it
• Encourage recycling of e-waste:
o Recovery of e-waste is abysmally low, we need to encourage recycling of e-
waste on a very large scale level so that problem of e-waste disposal is
contained.
• Viability gap funding:
o Government should provide VGF for waste processing infrastructure projects
to make in financially viable.

11 | iLearn IAS
• Replication of best practices such as Alapuzha model (Nirmala Nagaram, Nirmala
Bhavanam).

2.5. iTips:
• For prelims, questions are expected on Solid Waste Management Rules, GOBAR-
Dhan scheme etc.
• For mains, questions can be asked on the current scenario of waste management in
India, challenges faced in the SWM, steps taken by Government towards SWM etc.

12 | iLearn IAS
3. SHUKRAYAAN MISSION
3.1 WHY IN NEWS?
• The Indian Space Research Organisation (ISRO) has short-listed 20 space-based
experiment proposals for its proposed Venus orbiter mission 'Shukrayaan'.
3.2 SYLLABUS:
GS 3: Awareness in the fields of IT, Space.
3.3 PRACTICE QUESTION:
Q. How will the Venus orbiter Mission of ISRO help to understand the origin and evolution
of Earth-like planets?
3.4 ANALYSIS:
SHUKRAYAAN-1:
• Shukrayaan-1 is a proposed orbiter to Venus by the ISRO.
• The mission’s primary objectives are to map Venus’ surface and subsurface while
studying the planet’s atmospheric chemistry and interaction with the solar wind.
• In its current configuration, the orbiter weighs about 2,500 kilograms and will carry
science payloads like synthetic aperture radar. Russia, France, Sweden and Germany
will have instruments onboard.
• ISRO aims to launch the orbiter in late 2024. It aims to study the planet for more
than four years.
• Shukrayaan is currently slated to launch on India’s GSLV Mk II rocket. However, ISRO
is also evaluating the possible use of the more powerful GSLV Mk III rocket, which
would allow Shukrayaan to carry more instruments or fuel.
VENUS:
• Venus is the second planet from the Sun. It is named after the Roman goddess of
love and beauty.
• Venus orbit the Sun every 224.7 Earth days. It spins in the opposite direction of
Earth, with a rotation period of 243 Earth days, making it the longest among the
planets in our Solar System.
• Similar in size and structure to Earth, Venus has been called Earth's twin. However,
there are radical differences between the two worlds.
o Venus has a dense, toxic atmosphere filled with more than 96% carbon
dioxide and it is perpetually shrouded in thick, yellowish clouds of mostly
sulfuric acid that trap heat. As a result, Venus has the hottest surface of any
planet in the Solar System.
o Venus has crushing air pressure at its surface – more than 90 times that of
Earth
o Venus' solid surface is a volcanic landscape covered with extensive plains
featuring high volcanic mountains and vast ridged plateaus.

13 | iLearn IAS
• It is the second-brightest natural object in Earth's night sky after the Moon. But
Venus does not have any moons of its own.
• Venus was the first planet to be explored by a spacecraft – NASA’s Mariner 2. Since
then, more than 40 spacecraft have explored Venus. Eg: Magellan mission of ‘90s,
which mapped the planet's surface.
WHY EXPLORE VENUS?
• Presence of life: In September 2020, scientists announced that they have detected
phosphine molecules in Venus, which could be a biosignature of microbial life. The
idea that Venus used to be habitable like Mars warrants further exploration and can
have profound scientific implications.
• Presence of water in the past: Past missions to Venus have observed granite-like
rocks on the surface, which require abundant water to form.
• Understand earth’s evolutionary history: Studying Venus helps scientists get
answers to questions like how Venus transformed from a potentially habitable world
to its current state, which can provide insights into what makes Earth a habitable
planet. For Eg: Japan’s Akatsuki spacecraft discovered a giant stationary wave in
Venus’ atmosphere, (the largest of its kind in the solar system) which can provide
insights into the evolution of atmosphere.
• Predict trends in earth’s climate change: Venus helps scientists model Earth’s
climate, and serves as a cautionary tale on how dramatically a planet’s climate can
change.
• Develop comparative planetary studies: Scientists use Venus as a reference to
understand how Earth-sized planets around other stars evolve and what conditions
might exist there. For eg: The Magellan mission found no sign of plate tectonics on
Venus, which means that volcanoes there must work differently than on Earth.
• Aid future missions: Several scientists and engineers view Venus as a crucial part of
manned interplanetary missions. For Eg: They propose that a flight to/from Mars can
happen more quickly and cheaply if it involves using a gravity assisted slingshot past
Venus.
CHALLENGES AHEAD:
• Venus is a hostile planet to study: Its thick clouds make research from an orbiter
difficult, while heat, high pressure, and sulfuric acid droplets make studies on the
surface a technological nightmare. Hence, highly sophisticated and sensitive
technologies are needed for the orbiter.
• Longevity of orbiter: As per NASA, spacecrafts don't last long around Venus as its
high temperature overheats electronics in a very short time span. Hence, ISRO has to
develop new heat resistant materials and electronics for the mission to succeed.
• Limited collaboration opportunities: Of the more than 40 Venus missions so far,
roughly half have failed. Only three spacecraft have orbited Venus in the past 30

14 | iLearn IAS
years. Hence, India will have to design and develop everything indigenously from the
scratch.
CONCLUSION:
• ISRO’s success in the Mangalyaan and Chandrayaan missions provide an impeccable
track record mission for India in interplanetary missions. Also, the Aditya L1 mission
will provide an ideal platform for India to develop heat resistant instruments and
craft technologies. These factors provide an ideal environment for India to develop
upon the Venus mission.
• Space agencies around the world are showing renewed interest in the second planet
from the sun. NASA selected two Venus missions earlier this year for further
consideration for launch opportunities in 2025 and 2028. The European Space
Agency is considering a Venus orbiter mission called EnVision. Hence, India’s entry
into this foray is at the opportune time.
3.5. iTips:
• For Mains, the content can also be used in questions related to developments in
India's space technology.
• For Prelims, questions on the features of Venus, various international space missions
and other interplanetary missions can be asked.

15 | iLearn IAS
4. METHANOL ECONOMY PROGRAM
4.1 WHY IN NEWS?
In Sept 2020, Coal India floats tender to build country's first coal to methanol (C2M)
plant in West Bengal. The move is in line with the Niti Ayog’s push for developing a
methanol economy as one of the options to reduce India’s carbon footprint and the oil
import bill.
4.2 SYLLABUS:
GS 3: Infrastructure: Energy, Ports, Roads, Airports, Railways etc.
GS 3: Conservation, Environmental Pollution and Degradation, Environmental Impact
Assessment.
4.3 PRACTICE QUESTION:
Q. “Methanol and Dimethyl ether (DME) can play an important role in order to contain the
rising imports and improve the energy security of India”. Discuss
4.4 ANALYSIS:
BRIEF DESCRIPTION OF METHANOL AND DME:
• Methanol is a single carbon compound which can be produced from coal, natural
gas, biomass (i.e. products which are capable of producing syngas)
• It is light, volatile, colourless and flammable liquid with distinctive odour
• It burns with invisible flame and is bio-degradable
• It is highly toxic to humans if ingested
• Dimethyl ether (DME) is the simplest ether compound can be produced from
methanol or directly from syngas.
• It is gaseous version of methanol and can be blended with LPG. It is a non-toxic
compound
NEED FOR METHANOL PROMOTION:
• Ever-increasing demand for energy:
o Energy is considered as of the key inputs for economic development of the
country.
o India is poised to play a significant role in the Global energy space, as it is
likely to account for 25% of the rise in global energy demand by 2040
o Our country’s energy demand is expected to rise at a compounded annual
growth rate (CAGR) of 3.5% till 2040 as it advances on the path of
development4
• High import dependence:
o India imported 37% of its total primary energy demand in 2015-16
o Import dependence of crude oil and natural gas has increased from 73% and
17% in 2005-06 to 81% and 40% in 2015-16 respectively.
o Methanol Economy will result in minimum 15% of reduction in fuel bill
annually for the country by 2030

16 | iLearn IAS
• Dismal growth in domestic production of oil and gas:
o There has been a dismal growth in domestic oil (CAGR –1.4%) and natural
gas (0.01%) production over the last decade
• Cleaner fuel:
o Methanol is an efficient fuel (octane number 100) and emits lesser NOx and
Particulate matter (PM) than gasoline and produces no SOx as there is no
sulphur in methanol.
o DME is also an efficient fuel and burns with lesser NOx and PM
• Environmental issues:
o India is the 3rd largest emitter of CO2 >> thus needs to switch to cleaner fuels
to meet our INDC targets under Paris Agreement
• Worsening air quality in Indian cities:
o Methanol usage can bring down GHG emissions by 20% in terms of
particulate matter, NOx, and SOx thereby improving urban air quality.
• Coal availability:
o China is leading the world with the largest production of Methanol & DME.
China produces 70% of its methanol from Coal
o India can also follow the footsteps of China as our country has the 5th
largest coal reserves in the World
o Weak global coal prices and stricter environmental laws are likely to offer
firm coal to methanol margins
STATUS OF METHANOL IN INDIA:
• India is at a nascent stage in methanol production and usage, but it has a large
potential given its wide applications
• There are 5 main producers of methanol in India - Assam Petrochemicals, National
Fertilizers Limited etc.
• Domestic production of methanol has fallen by 57% from 2010-11 to 2015-16,
whereas the consumption has risen by 61% over the same period
• There has been continuous increase in methanol imports in India as they have more
than doubled from 2010-11 to 2015-16
• 90% of methanol requirement is met through imports. This is primarily because, it is
cheaper for India to import methanol in comparison with domestic production.
• India imports 99% of its methanol from Iran and Saudi Arabia, where methanol is
produced from natural gas which is abundantly available in latter countries at
extremely low prices
OPPORTUNITIES/ADVANTAGES:
• Methanol and DME to be used as a Transportation fuel:
o Road sector:
▪ Methanol & DME can be blended with gasoline and diesel, or can
completely substitute the latter fuels respectively giving us an
opportunity to reduce our dependence on imported crude oil.

17 | iLearn IAS
▪ India has already set itself an ambitious target of 10% reduction in
import dependence of oil & gas by 2022
▪ High methanol blends offer significant vehicle efficiency improvement
– potential of 25%.
o Railways:
▪ This also offers an opportunity for the railway engines to run on
methanol/DME blends.
o Waterways:
▪ India envisages to roll out a massive water transportation system
under the aegis of its flagship Sagarmala project
▪ In order to check the pollution caused by diesel run ships, methanol
and DME powered ships would not only be cost effective alternatives
but would also produce far less pollution.
• Help in achieving the objective of access to clean cooking fuels:
o India houses nearly 800 million people without access to clean cooking fuels
which largely rely on biomass to meet their cooking requirements.
o Government launched an initiative, Pradhan Mantri Ujjwala Yojana (PMUY)
in 2016 under which 5 Cr LPG connections will be distributed to Below
Poverty Line (BPL) households
o India imported 46% of its LPG requirements in 2015-16 and its imports are
only going to rise in the near future.
o Therefore, methanol or DME blending with LPG or the complete
substitution of latter through former can not only gradually displace LPG
imports, but would also help in enhancing the access to clean cooking fuels in
India
• Displacing diesel in Telecom Towers:
o A large number of telecom towers, especially in rural areas run on diesel for
as long as 18-20 hours a day because of frequent electric cuts.
o Telecom towers in India consume around 2% of diesel consumption which is
a significant amount indicating a vast potential for DME to replace diesel.
• Production of various chemicals:
o Methanol can be used for producing various chemicals like formaldehyde,
acetic acid and olefins which can be exported and can be high foreign
exchange earners
• Dovetailing with Swachh Bharat Mission:
o Apart from coal, biomass/Municipal Solid Waste (MSW) to methanol can
also be a viable option for India which can be dovetailed with Swachh Bharat
Mission.
o The current availability of biomass in India is estimated to be in the range of
500-650 MT, however, a proper supply chain mechanism has to be created

18 | iLearn IAS
for the same so that there is a continuous availability of biomass for
methanol production.
o Moreover, it can be an opportunity for India to use its landfills to convert it
into methanol and avoid problems such as toxins leaching into the soil and
release of GHG emissions etc.
• Rising market:
o The market for methanol is expected to register a CAGR of 5.64% during the
forecast period (2019-2024).
o One of the major factors driving the market studied is the increasing demand
for methanol-based fuel.
• Employment opportunities:
o It can create close 5 million jobs through methanol production/application or
distribution services
CHALLENGES:
• May not be financially attractive in the current scenario:
o Investment in methanol/DME production might not look much attractive
amidst the low global crude prices which have been a huge relief for India
• Reliance on natural gas:
o India relies on imported natural gas for methanol production due to which it
loses its competitiveness in comparison with imports
• Well to wheel (WTW) emissions:
o The tailpipe emissions from methanol usage (i.e. at the consumption end)
are quite low in comparison with conventional fuels like gasoline and diesel,
however, the well to wheel (WTW) emissions for coal to methanol
production in comparison with gasoline are more
• Corrosive and highly toxic:
o Methanol is more corrosive than gasoline and may require new equipment
for storage and distribution of the same and is also toxic to humans if
ingested
INITIATIVES TAKEN:
• Task force constituted:
o A task force has been constituted which would work towards the
development of overall framework of Methanol production, distribution and
utilization in the country.
• Target based approach:
o The Government is likely to go ahead with a target of 15% blending by
methanol/DME in gasoline/diesel by 2022
• Coal based methanol plant:
o Coal India Limited (CIL) has unveiled its plan to set up a coal based methanol
plant in West Bengal and has even invited bids from licensors of coal
gasification technology to set up a coal to methanol plant

19 | iLearn IAS
• BIS has notified 20% DME blending with LPG:
o To the consumer, it will result in saving of minimum Rs. 50 to Rs. 100 per
cylinder.
• Pollution free cooking medium
o Assam petro-chemicals launched Asia’s first canisters based methanol
cooking fuel programme
• Methanol as marine fuel:
o Cochin Shipyard had made 3 boats and 7 cargo vessels for Inland Waterways
Authority of India (IWAI), that can run on methanol fuel
• Railways:
o In railways sector, RDSO is working towards blending of methanol in the
range of 5-20% through direct fuel injection in locomotives.
• International cooperation:
o Under Indian Methanol Economy program 5 methanol plants based on high
ash coal, 5 DME plants and 1 natural gas based methanol production plant
in joint venture with Israel, are planned to be set up.
SUGGESTIONS (as per NITI Aayog):
• Use coal instead of natural gas for production of methanol:
o Currently India is producing all of its methanol from imported natural gas, it
must use coal for methanol production which is expected to make it
economically viable to produce methanol in India
• Waste to energy:
o Apart from using coal as a feedstock, biomass/municipal solid waste and
flared natural gas can also be used for methanol production, but the
continuous availability of latter would be a challenge
o Generate methanol from waste dumped in landfill, using gasification
technology
• Creation of an innovation fund:
o The first and foremost step should be create an innovation fund that will
support the R&D activities for methanol/DME in India.
• State supported plant of coal to methanol production:
o Establishment demonstration plant of coal to methanol production in India.
(Coal India floats tender to build country's first coal to methanol (C2M) plant
in West Bengal)
o It is necessary to have a sufficient amount of methanol production capacity
in India so that the user industries are assured of supply.
• Promotion of flexi-fuel vehicles
o Development of flexi-fuel vehicles which would be able to run on
methanol/DME fuel blends
• For cleaner cooking:

20 | iLearn IAS
o A separate program for the development of methanol/DME cook-stoves can
be launched.
• Reduce carbon emission in railways:
o Program can be launched for converting diesel powered railway locomotives
to methanol/DME based engines
• International cooperation:
o India should also look at options to set up a manufacturing facility for
methanol/DME in Iran or Qatar as both these countries having huge reserves
of natural gas can provide the same at very low prices.
o Methanol/DME produced abroad can be imported in India for its direct
application or for further conversion to chemicals like olefins.
o Though, India may ending up importing methanol in the above scenario, it is
likely to be economically advantageous rather than importing crude.
• Mega coal-based complex:
o India must set up a mega coal based complex for production of power,
methanol and fertilizer in an integrated manner which would significantly
reduce the cost of various commodities produced.
BEST PRACTICES:
• Consumption of methanol in China has seen a rapid expansion at a CAGR of 18%
over the last decade.
• In 2016, China blended around 21 MT of methanol with gasoline, whereas India has
not started using methanol as a transportation fuel.
• Let alone, India’s gasoline consumption was 22 MT in 2015-16 which is
approximately equal to the amount of methanol blended with gasoline in China
• Moreover, 90% of the total DME production in China goes into LPG blending, where
DME is permitted as 20% blend on a weight basis
• This shows that China has been aggressively pursuing the agenda of alternative
fuels, especially using methanol to reduce its import dependence
• Moreover, a Chinese auto manufacturing company, Geely, has already
commissioned methanol fueled automobile plant which will manufacture two lakh
cars annually
CONCLUSION
• India can leapfrog to a Methanol Economy before the world including India
transitions to a hydrogen economy in the long term.
4.5. iTips:
• For prelims, basic features of Methanol and Dimethyl ether (DME), status methanol
production in India etc.
• For mains, questions can be asked on prospects and challenges of methanol
economy, initiatives taken to promote methanol etc.

21 | iLearn IAS
5. DEEP OCEAN MISSION
5.1 WHY IN NEWS?
India will soon launch its ambitious ‘Deep Ocean Mission’ that envisages exploration
of minerals, energy and marine diversity of the underwater world. The mission will likely
cost the exchequer Rs 4,000 crore.
5.2 SYLLABUS:
GS 3: Achievements of Indians in science & technology; indigenization of technology and
developing new technology.
5.3 PRACTICE QUESTION:
Q. Examine the prospects of deep ocean resources with special reference to India’s Deep
Ocean Mission?
5.4 ANALYSIS:
DEEP SEA MISSION:
• It is the Government of India mission to study the various aspects of ocean in an
integrated framework, as the Indian Space Research Organisation (ISRO) has been
studying the space.
• The multi-disciplinary work will be piloted by the Ministry of Earth Sciences and
other government departments like the DRDO, Dept of Biotechnology, ISRO and
CSIR.
• The mission is expected to cost over ₹4,000 crore and is part of the umbrella scheme
of the Ministry of Earth Sciences - “Ocean Services, Technology, Observations,
Resources Modelling and Science (O-SMART)”.
• Its major objectives include:
o Design, development and demonstration of human submersibles
o Explore the possibility of deep-sea mining and developing necessary
technologies to harness various resources from the seabed and deep water.
o Undertaking ocean floor survey and energy exploration.

• To support these efforts, India had signed a 15-year contract with the International
Seabed Authority (ISA) in 2016 for exploration of Poly-Metallic Sulphides (PMS) in

22 | iLearn IAS
the Indian Ocean. Based on this, India has ear-marked nearly 1.5 lakh square
kilometres of area in the central Indian Ocean for exploration.
O-SMART:
• It is an umbrella scheme under the Ministry of Earth Sciences which encompasses a
total of 16 sub-projects addressing ocean development activities.
• The important deliverables envisaged under the scheme include:
1. Strengthening of Ocean Observations and Modelling
2. Strengthening of Ocean Services for Fishermen
3. Setting up Marine Coastal Observatories for monitoring marine pollution
4. Setting up Ocean Thermal Energy Conversion Plant (OTEC) in Kavaratti
5. Acquisition of 2 Coastal Research Vessels for Coastal research
6. Continuation of Ocean Survey and Exploration of Minerals and Living
Resources
7. Technology Development for Deep Ocean Mining- Deep Mining System and
Manned Submersibles
8. Setting up Six Desalination Plants in Lakshadweep
• The services rendered under the O-SMART will provide economic benefits to a
number of user communities in the coastal and ocean sectors, namely, fisheries,
offshore industry, coastal states, defence, shipping, ports etc.
• Implementation of O-SMART will help in addressing issues relating to Sustainable
Development Goal-14, which aims to conserve use of oceans, marine resources for
sustainable development.
• This scheme (O-SMART) also provides necessary scientific and technological
background required for the implementation of various aspects of Blue Economy.
DEEP SEA RESOURCES:
• Resources such as fish, oil and gas have been taken from the sea for many decades.
• More recently the marine environment has been targeted as a source of renewable
energy. Eg: Ocean thermal energy conversion to produce electricity for desalination.
• The most significant targets for latest deep-sea mining efforts are polymetallic
sulphides, manganese nodules and cobalt-rich ferromanganese crusts.
o Poly-Metallic Sulphides (PMS) are precipitates of hot fluids from upwelling
hot magma from deep interior of the oceanic crust, discharged through
mineralized chimneys.
o Polymetallic manganese nodules (PMN) are small potato-sized lumps of
material precipitated from seawater and sediment pores at slow rates. They
contain approximately 24% manganese, 14% iron and some traces of copper.
o Cobalt-rich ferromanganese crusts occur in areas of significant volcanic
activity. The crusts grow on hard-rock substrates of volcanic origin by the
precipitation of metals dissolved in seawater.

23 | iLearn IAS
• On a longer run, there will be interest in the potential extraction rare earth elements
(REEs) in deep-sea muds and gas hydrates in the ocean floor. Gas hydrates are a
crystalline solid formed of water and gas. It looks and acts much like ice, but it
contains huge amounts of methane.
SIGNIFICANCE:
• Economic benefits: The mission will help in leveraging the idea of blue economy for
the country’s overall economic growth. It will also give a boost to efforts of exploring
India’s vast Exclusive Economic Zone and Continental Shelf.
• Large unexplored reserves: Minerals and energy diversity in the underwater world
forms a big portion, most of which remains unexplored even today. By developing
extraction technology and capability, India can become a pioneer in mineral
extraction in the future.
• Resource security: India has shortage of vital mineral resources such as copper and
rare earth minerals, which are abundant in the polymetallic nodules. Also,
developments in ocean based renewable energy can strengthen India’s energy
basket.
• Sustainable development: Sustainable resource extraction from land-based sources
is proving to be a major challenge. Also, SDG 14 explicitly calls for sustainable use of
marine resources. In this regard, the deep-sea mission seeks to develop sustainable
solutions.
• Develop indigenous capabilities: At current levels, extraction of deep-sea resources
is not economical. In this light, the mission involves developing different deep ocean
technologies. If India can develop more efficient and economical extraction
technologies,
• Strategic advantage:
o Global: Deep sea mineral exploration and use of deep-sea resources is one
area where there are no clear international legislations or guidelines.

24 | iLearn IAS
Through the mission, India aims is to be prepared when such rules are
formalized.
o Regional: The move is strategically significant as it will enhance India’s
presence in the Indian Ocean where other players like China, Korea and
Germany are active.
CONCERNS:
•Global commons: The global commons include those parts of the Earth's surface
beyond national jurisdictions — notably the outer space, Antarctic, open ocean and
the living resources found there. Sustainable extraction and equitable sharing of
such resources will require an effective legislation, which is currently lacking.
• Economic viability of mining: The latest estimate from the ISA says it will be
commercially viable only if about three million tonnes are mined per year. But to
attain such levels, more studies are being carried out and technology needs to be
scaled up rapidly.
• Disturbance of the seafloor: The scraping of the ocean floor can alter or destroy
deep-sea habitats, leading to the loss of species and fragmentation or loss of
ecosystem structure and function. Many species living in the deep sea are endemic
and physical disturbances can possibly wipe out an entire species.
• Sediment plumes: Some forms of deep-sea mining will stir up fine sediments on the
seafloor, creating plumes of suspended particles. It is unclear how far these particles
may disperse beyond the mining area, how long it would take for them to resettle on
the seafloor, and to what extent they may affect ecosystems and species.
• Pollution: Species such as whales, tuna and sharks could be affected by noise,
vibrations and light pollution caused by mining equipment and surface vessels, as
well as potential leaks and spills of fuel and toxic products.
CONCLUSION:
The unsustainable terrestrial resource exploitation has led to a shortage of land
resources. This has forced us to venture into uncharted territories deep within the ocean to
meet our future resource demands. Though India is a pioneer in this field, we should take
care to see that the resources are extracted and used in a sustainable manner with
minimum impacts on the deep-sea environment.
5.5. iTips:
• For Prelims, questions on the various deep-sea resources, their origin, distribution
and features of ISA can be asked.
• For Mains, the content can also be used in questions related to blue economy,
energy crisis, India's achievements in science and technology and strategic efforts in
Indian ocean.

25 | iLearn IAS
WEEKLY PRELIMS COMPILATION

26 | iLearn IAS
Pushkarulu
Why in news?
o Tungabhadra Pushkaralu is being celebrated in Andhra Pradesh.
About Pushkarulu:
o Pushkaram is an Indian festival dedicated to worshiping of rivers. It is also
known as Pushkaralu (in Telugu), Pushkara (in Kannada) or Pushkar.
o It is celebrated at shrines along the banks of 12 major sacred rivers in India, in
the form of ancestor worship, spiritual discourses, devotional music and
cultural programmes.
o The celebration happens annually, once in 12 years along each river.
o Each river is associated with a zodiac sign, and the river for each year's festival
is based on which sign Jupiter is in at the time.
o Due to regional variations, some of the zodiac signs are associated with
multiple rivers.
o Rivers having pushkarulu include Sindhu/Indus, Ganga, Yamuna, Sarasvati,
Narmada, Cauvery, Tapti, Godavari, Krishna, Tungabhadra, Pranahita and
Bhima.
PRELIMS QUESTION
1.Through which of the following states does Tungabhadra River flow through:
1. Karnataka
2. Maharashtra
3. Andhra Pradesh
4. Telangana
Select the correct answer using the code given below:
(a) 1 and 3 only
(b) 2,3 and 4 only
(c) 1,3 and 4 only
(d) 1,2,3 and 4 only
Answer: C

Inner line Permit


Why in news?
o Demonstration in Meghalaya for implementation of Inner Line Permit.
What is an Inner Line Permit?
o An Inner Line Permit is an official travel document required by Indian citizens to
visit or stay in states and regions where it is applicable.
o Arunachal Pradesh, Mizoram and Nagaland (except the city of Dimapur) are
currently under the ILP system, and Indian citizens who do not belong to these
regions need to have the document to visit or stay.

27 | iLearn IAS
o Outsiders cannot overstay the timeline mentioned in their permits.
o Different types of permits are provided — for tourists, for tenants and for other
employment purposes, based on the period of stay.
o The system is an offshoot of the colonial-era Bengal Eastern Frontier
Regulations, 1873.
o The British had brought in these regulations to protect the commercial interest
of the crown by regulating entry and exit of Indian citizens from outside these
regions for the purpose of trade.
o After Independence, the Government of India chose to keep the ILP to protect
the interests of the indigenous tribal communities of the Northeast. The words,
“British subjects (Indians)”, in the original regulations were replaced by “Citizen
of India”.
PRELIMS QUESTION
2.Which of the following states currently have an inner line permit system in
place?
1. Arunachal Pradesh
2. Mizoram
3. Nagaland
4. Meghalaya
Select the correct answer using the code given below:
(a) 1 and 2 only
(b) 1 and 3 only
(c) 1,2,3 only
(d) 1,2,3,4
Answer: C

Deemed Forests
Why in news?
o Karnataka forest ministed announced in the Assembly that the state
government would soon declassify 6.64 lakh hectares of the 9.94 lakh hectares
of deemed forests in the state (nearly 67%) and hand it over to Revenue
authorities.
o The move has been taken after a study of the actual extent of deemed forest
areas by local committees headed by officials from the Revenue, Forest and
Land Records Departments in every district.
About deemed forests:
o The issue of deemed forests is a contentious one in Karnataka, with legislators
across party lines often alleging that large amounts of agriculture and non-
forest land are “unscientifically” classified as such.

28 | iLearn IAS
o While the concept of deemed forests has not been clearly defined in any law
including the Forest Conservation Act of 1980, the Supreme Court in the case
of T N Godavarman Thirumalpad (1996) accepted a wide definition of forests
under the Act.
o An expert committee constituted by the Karnataka government after the
Supreme Court order identified ‘deemed forests’ as “land having the
characteristic of forests irrespective of the ownership’”; Thickly wooded areas
of the Revenue Department not handed over to the Forest Department; thickly
wooded areas recommended to be handed over to the Forest Department;
thickly wooded land distributed to grantees but not cultivated; and thickly
wooded plantations of the Forest Department could all be ‘deemed forests’.
Demands to reclassify:
o Reports by expert committees in 1997 and 2002 identified 43.18 lakh hectares
of forest land for conservation in Karnataka, which included 33.23 lakh hectares
notified forest area as per forest records and 9.94 lakh hectares ‘deemed
forests.
o Due to criticism that the classification move hit farmers, as well as barred large
tracts from mining, the state has been arguing that the classification was done
without taking into account needs of people.
o Since then government has looked at the issue and decided the amount of
deemed forest land to be released as 6.64 lakh hectares.
o Since Preservation of forest areas in India under the Forest Conservation Act,
1980 has been continuously monitored by the Supreme Court since the
Godavarman case judgment in 1996, the state government must obtain
clearances from the Supreme Court for affecting changes to land classified as
deemed forests.
PRELIMS QUESTION
3.Which of the following conservation areas are declared in India as per Wildlife
protection act?
1. Wildlife Sanctuary
2. Community reserve
3. Eco-Sensitive Zones
4. Coastal regulation zones
Select the correct answer using the code given below:
(a) 1 and 2 only
(b) 1 and 3 only
(c) 1,2,3 only
(d) All of the above
Answer: A

29 | iLearn IAS
Brahmaputra River
Why in news?
o The proposed 18 Km-long bridge over the Brahmaputra river, connecting Assam
with Meghalaya, will be constructed soon and will be the longest river bridge in
India.
About the River Course:
o The Brahmaputra called Yarlung Tsangpo in Tibet, Siang/Dihang River in
Arunachal Pradesh, is a trans-boundary river which flows through Tibet, India
and Bangladesh.
o It is the ninth largest river in the world by discharge, and the 15th longest.
o With its origin in the Manasarovar Lake region, near the Mount Kailash, it flows
along southern Tibet to break through the Himalayas in great gorges and into
Arunachal Pradesh (India).
o It flows southwest through the Assam Valley as Brahmaputra and south
through Bangladesh as the Jamuna (not to be mistaken with Yamuna of India).
o In the vast Ganges Delta, it merges with the Padma, the popular name of the
river Ganges in Bangladesh, and finally, after merging with Padma, it becomes
the Meghna and from here, it flows as Meghna river before emptying into the
Bay of Bengal.
o Major Left Bank tributaries include Lhasa River, Nyang River, Parlung Zangbo,
Lohit River, Dhansiri River, Kolong River
o Major right bank tributaries are: Kameng River, Manas River, Beki River,
Raidak River, Jaldhaka River, Teesta River, Subansiri River
Navigation and Ecology:
o About 4,696 km long, the Brahmaputra is an important river for irrigation and
transportation in the region.

30 | iLearn IAS
o The river is prone to catastrophic flooding in the Spring when the Himalayan
snow melts.
o It is a classic example of a braided river and is highly susceptible to channel
migration and avulsion.
o It is also one of the few rivers in the world that exhibits a tidal bore. It is
navigable for most of its length.
o National Waterway 2 (NW2) is 891 km long Sadiya-Dhubri stretch of
Brahmaputra River in Assam.
o Its delta is home to 130 million people and 600 000 people live on the riverine
islands (the largest of which is Majuli, the first island to be made a district in
India).
PRELIMS QUESTION:
4.Which of the following are tributaries of River Brahmaputra?
1. Subarnarekha
2. Manas
3. Teesta
4. Chindwin
Select the correct answer using the code given below:
(a) 1 and 2 only
(b) 2 and 3 only
(c) 1,2,3 only
(d) All of the above
Answer: B

Barikot Ghundai
Why in news?
o An ancient Hindu Temple has been discovered at Barikot Ghunai in Pakistan.
About Barikot Ghundai:
o A Hindu temple, believed to have been constructed 1,300 years ago, has been
discovered by Pakistani and Italian archaeological experts at a mountain in
northwest Pakistan’s Swat district.
o The discovery was made during an excavation at Barikot Ghundai.
o The Temple discovered seems to be dedicated to Lord Vishnu.
o It was built by the Hindus 1,300 years ago during the Hindu Shahi period.
o The Hindu Shahis or Kabul Shahis (850-1026 CE) was a Hindu dynasty that ruled
the Kabul Valley (eastern Afghanistan), Gandhara (modern-day Pakistan), and
present-day northwestern India.
o During their excavation, the archaeologists also found traces of cantonment
and watchtowers near the temple site.

31 | iLearn IAS
o The experts also found a water tank near the temple site which they believe
was used by the Hindus for bathing before worship.
o This was the first temple of the Gandhara civilisation discovered in Swat
district.
Gandhara civilisation
o The Gandhara Civilization existed in what is now Northern Pakistan and
Afghanistan from the middle of the 1st millennium BCE to the beginning of the
2nd millennium CE.
o Multiple major powers ruled over this area during that time, most of whom
adopted and patronised the Indo Greek artistic tradition which had developed
in the region following Alexander’s invasions into India.
o Prominent dynasties who ruled the area include Mauryas, Scythians, Indo-
greeks, Kushanas, Huns and Hindu Shahis.
o The area was home to cities like Takshasila (Taxila), Purushapura (Peshawar)
and Pushkalavati (Mardan).
o There was a proliferation of stupas and other associated religious
establishments such as monasteries during this period in this region.
PRELIMS QUESTION
5.Consider the following Buddhist archaeological sites and their locations:
1. Thotlakonda : Andhra Pradesh
2. Lalitagiri : Odisha
3. Kanaganahalli : Karnataka
Select the correct answer using the code given below:
(a) 1 and 2 only
(b) 1 and 3 only
(c) 2 and 3 only
(d) All of the above
Answer: D

Roshni Act
Why in news?
o The Jammu and Kashmir administration on Monday published on its website
the list of beneficiaries under the Roshni Act.
o The beneficiaries include former State ministers, retired civil servants and
politicians.
About Roshni Act:
o Jammu and Kashmir States Land (vesting of ownership to the occupants) Act,
also known as Roshini Act was a land law in erstwhile Jammu and Kashmir
State.

32 | iLearn IAS
o The Roshni Act envisaged the transfer of ownership rights of state land to its
occupants, subject to the payment of a cost, as determined by the government.
o It was enacted by Chief Minister Farooq Abdullah’s government in 2000, and it
set 1990 as the cut off for encroachment on state land.
o The government’s target was to earn Rs 25,000 crore by transferring 20 lakh
kanals of state land to existing occupants against payment at market rates.
o The government said the revenue generated would be spent on
commissioning hydroelectric power projects, hence the name “Roshni”.
o Under later amendments the cut off was relaxed further to 2007.
o The government also gave ownership rights of agricultural land to farmers
occupying it for free, charging them only Rs 100 per kanal as documentation
fee.
Why it was repealed?
o Investigations into the land transfers subsequently found that land in Gulmarg
had been given over to ineligible beneficiaries.
o In 2009, the State Vigilance Organisation registered an FIR against several
government officials for alleged criminal conspiracy to illegally possess and vest
ownership of state land to occupants who did not satisfy criteria under the
Roshni Act.
o The Comptroller and Auditor General, in a 2014 report, had stated that only
₹76 crore had been realised from the transfer of encroached land between
2007 and 2013, as against the target of about ₹25,000 crore.
o In November 2018, the High Court restrained all beneficiaries of the Roshni
scheme from selling or carrying out any other transaction in respect of the land
transferred to them.
o The J&K government on October 31, 2020 decided to declare all the actions
taken under the Roshni Act, as “null and void”, and has decided to retrieve the
land within six months.
PRELIMS QUESTION
6.Consider the following statements:
1. Hemis national park is located in the Union territory of Jammu and Kashmir.
2. Wular Lake located in Jammu and Kashmir is designated to be of
international importance under the Ramsar Convention.
Which among the above statements is/are correct?
(a) 1 only
(b) 2 only
(c) Both 1 and 2
(d) Neither 1 nor 2
Answer: B

33 | iLearn IAS
Swiss Challenge method
Why in news?
o Brihanmumbai Municipal Corporation (BMC) is planning to take the ‘Swiss
Challenge Method’ for creating Mumbai’s first desalination plant.
o The proposed plant, which will be able to desalinate 200 million litres of water
day by day (MLD), will come up at Manori and will help overcome the water
scarcity confronted by Mumbai.
About Swiss challenge method:
o Swiss Challenge Method is a method of bidding, often used in public projects,
in which an interested party initiates a proposal for a contract or the bid for a
project.
o The government then puts the details of the project out in the public and
invites proposals from others interested in executing it.
o On the receipt of these bids, the original contractor gets an opportunity to
match the best bid.
Why is it important?
o The Swiss Challenge allows a seller to mix-and-match the features of both an
open auction and a closed tender to discover the best price for an asset.
o In its original form, a Swiss Challenge allows an infrastructure developer to
come up with a suo motu proposal for a new project without waiting for the
government to call for bids.
o This can foster innovation, as contractors or developers may initiate projects
that the powers-that-be didn’t even think of.
o The method was upheld by the Supreme Court of India for awarding public
projects and the Government of India has tried out this method in road and
railway projects.
What are the problems?
o By allowing a bidder to initiate an idea and giving him the first right of refusal,
the Swiss Challenge can promote favouritism in the award of public projects,
opening the doors to corruption.
o To guard against this, legal experts suggest an open list of public projects that
allow Swiss Challenge and full public disclosure of bid details when the
government receives a proposal.
PRELIMS QUESTION
7.Consider the following statements about the revamped Viability Gap Funding
(VGF) scheme:
1. It has a budget of more than 8000 crore rupees.
2. Funding under the scheme is limited to projects concerning economic
infrastructure.

34 | iLearn IAS
Which among the above statements is/are correct?
(a) 1 only
(b) 2 only
(c) Both 1 and 2
(d) Neither 1 nor 2
Answer: A

Pangda Village and Doklam


Why in news?
o Chinese media claimed that a new border village built by China near Bhutan
was on Chinese territory but released images of the village show its location on
territory disputed by the two countries.
o The Village is called Pangda and holds around 27 houses.
o An image released by the newspaper placed the village in disputed territory, a
couple of kilometres inside what Bhutan sees as its territory.
About Doklam:
o Doklam, or Donglang in Chinese, is an area spread over less than a 100 sq km
comprising a plateau and a valley at the trijunction between India, Bhutan
and China.
o It is surrounded by the Chumbi Valley of Tibet, Bhutan’s Ha Valley and Sikkim.
o The region has been an area of dispute between China and Bhutan and has not
been resolved.
o It flared up in 2017 when the Chinese were trying to construct a road in the
area, and Indian troops, in aid of their Bhutanese counterparts, objected to it,
resulting in the stand-off.
o Doklam is strategically located close to the Siliguri Corridor, which connects
mainland India with its north-eastern region. The corridor, also called Chicken’s
Neck, is a vulnerable point for India.
Why does it matter?
o While India-Tibet trade flourished along the Siliguri corridor and Chumbi Valley,
Doklam had very little significance.
o Even during British rule, Doklam did not command much attention. In recent
years however, China has been beefing up its military presence in the Chumbi
Valley, where the Chinese are at a great disadvantage militarily.
o Both Indian and Bhutanese troops are on a higher ground around the Valley.
o Control over Doklam would give Chinese a commanding view of and an easy
access to both the Chumbi Valley and the Siliguri Corridor.
o The desolate Doklam region grabbed global attention after the 2017 stand-off
when Indian troops entered Doklam, at the request of Bhutan, and stopped the
construction of a road Chinese were constructing inside Bhutanese Territory.
35 | iLearn IAS
PRELIMS QUESTION
8.Consider the following Trijunctions between India and neighbouring countries:
1. Lipu Lekh Pass: Bangladesh- Myanmar
2. Jongsong Peak: Nepal-China
3. Diphu Pass: Myanmar - China
Which of the pairs given above is/are correct?
(a) 1 and 2 only
(b) 1 and 3 only
(c) 2 and 3 only
(d) 1,2,3 only
Answer: C

Unified Mobile Application for New-age Governance


(UMANG)
Why in news?
o To mark the occasion of 3 years of UMANG and 2000 plus services milestone,
an online conference was organized under the chairmanship of Minister for
Electronics and Information Technology, Ravi Shankar Prasad.
What is UMANG?
o UMANG is developed by National e-Governance Division (NeGD), Ministry of
Electronics and Information Technology (MeitY).
o UMANG provides a single platform for all Indian Citizens to access pan India e-
Gov services ranging from central to local govt bodies and other citizen centric
services.
o The app is for everyone who needs to interact with any government or semi-
government depart at center, state or local level in India.
o It makes available online a plethora of off-line services through a single unified
app. Customer Support is available from 8 am to 8 pm all days of the year.
o It has synergies with other key Government of India schemes, such as Digital
India, National e-Governance Plan, BharatNet, Make in India, Startup India,
Standup India, Industrial corridors, Bharatmala, Sagarmala, Dedicated Freight
Corridors and UDAN-RCS.
International Version of UMANG:
o The minister of IT has launched the UMANG’s international version in
coordination with Ministry of External Affairs.
o It can be used in select countries that include USA, UK, Canada, Australia, UAE,
Netherlands, Singapore, Australia and New Zealand.
o It will help Indian international students, NRIs and Indian tourists abroad, to
avail Government of India services, anytime.

36 | iLearn IAS
o It will also help in taking India to the world through ‘Indian Culture’ services
available on UMANG and create interest amongst foreign tourists to visit India.

PRELIMS QUESTION
9.Consider the following Mobile apps and the sectors with which they are
associated:
1. Tarang : Power Transmission Sector
2. SAARTHI : Highway Construction
3. Aapoorti : Railway procurement
Which of the pairs given above is/are correct?
(a) 1 and 2 only
(b) 1 and 3 only
(c) 2 and 3 only
(d) All of the above
Answer: B

Sentinel-6 satellite
Why in news?
o The first of Two Satelites in sentinel mission, Sentinel-6 Michael Freilich
satellite was launched recently.
About the Satellite:
o The Copernicus Sentinel-6 mission is a collaboration of the European
Commission, the European Space Agency (ESA), EUMETSAT, NASA and the US
National Oceanic and Atmospheric Administration (NOAA).

37 | iLearn IAS
o Sentinel-6a will be the first of two identical satellites -- the second to be
launched in five years -- that will provide measurements of unprecedented
precision until at least 2030.
o The twin satellites will measure sea-level rise, tracking changes threatening to
disrupt tens of millions of lives within a generation.
o Each Sentinel-6 probe carries a radar altimeter, which measures the time it
takes for radar pulses to travel to Earth’s surface and back again.
o They are designed to last for five-and-a-half years but could provide data for far
longer.
o The Sentinel-6 mission is part of the Copernicus programme initiative which is
European Union's Earth observation programme coordinated and managed by
the European Commission in partnership with the European Space Agency
(ESA), the EU Member States and EU Agencies.

PRELIMS QUESTION
10.Consider the following statements:
1. Thermal expansion of sea water due to global warming is a major cause of
Global sea level rise.
2. Global sea level rise will not be uniform everywhere on Earth.
Which among the above statements is/are correct?
(a) 1 only
(b) 2 only
(c) Both 1 and 2
(d) Neither 1 nor 2
Answer: C

Doddagaddavalli hoysala Temple


Why in news?
o Miscreants have desecrated and broken the idol of goddess Mahakali at the
historic Hoysala era Lakshmi temple at Doddagaddavalli
38 | iLearn IAS
About the temple:
o The Lakshmi Devi temple is located in Doddagaddavalli, a village in Hassan
District of Karnataka.
o The Lakshmi Devi temple, was built by the Hoysala Empire King
Vishnuvardhana in 1114 C.E.
o It is one of the earliest known temples built in the Hoysala style and the
building material is Chloritic schist, more commonly known as soapstone.
o The temple does not stand on a jagati (platform), a feature which became
popular in later Hoysala temples.
Other famous Hoysala temples:
o Chennakesava Temple at Belur dedicated to Vishnu
o Hoysaleswara temple at Halebidu dedicated to Shiva
o The Kesava temple at Somanathapura dedicated to Vishnu
o Akkana Basadi Jain Temple at Shravanabelagola
Hoysala Architecture:
o Hoysala rulers were influenced by the Western Chalukyan architecture and
demonstrates a distinct style different from Dravidian architecture of Tamil
Regions.
o An abundance of figure sculpture covers almost all the Hoysala temples using
episodes from mythology as themes.
o The Soapstone, which allows fine detailing and clarity was used as building
material to enable the sculptures.
o Hoysala temples are sometimes built on a raised platform or jagati which is
used for the purpose of a pradakshinapatha.
o They are also noted for their vimanas which are either stellate, semi–stellate
or orthogonal in plan.

PRELIMS QUESTION
11.Consider the following pairs of temples and the dynasties which built them:
1. Kanchi Kailasanathar Temple : Cholas

39 | iLearn IAS
2. Pattadakkal Virupaksha Temple : Chalukyas
3. Puri Jagannatha Temple : Eastern Gangas
Which of the pairs given above is/are correct?
(a) 1 and 2 only
(b) 1 and 3 only
(c) 2 and 3 only
(d) All of the above
Answer: C

Lachit Borphukan
Why in news?
o PM Modi and Union Home Minister Amit Shah paid tribute to Lachit
Borphukan, a celebrated warrior of Assam.
About Lachit Borphukan:
o Lachit Borphukan was a commander in the Ahom kingdom of medieval
Assam.
o He is known for his leadership in the 1671 Battle of Saraighat that thwarted a
drawn-out attempt by Mughal forces to gain Control over Brahmaputra Plains.
o On 24 November each year Lachit Divas (Lachit Day) is celebrated statewide in
Assam to commemorate the heroism of Lachit Borphukan and the victory of
the Assamese army at the Battle of Saraighat.
Battle of Saraighat:
o The Battle of Saraighat was a naval battle fought in 1671 between the Mughal
Empire (led by the Kachwaha king, Raja Ramsingh I), and the Ahom Kingdom
(led by Lachit Borphukan).
o The battle was fought on the Brahmaputra river at Saraighat, now in
Guwahati.
o Ahom Army defeated the Mughal Army by brilliant uses of the terrain, clever
diplomatic negotiations to buy time, guerrilla tactics, psychological warfare,
military intelligence and by exploiting the sole weakness of the Mughal forces—
its navy.
o The Battle of Saraighat was the last battle in the last major attempt by the
Mughals to extend their empire into Assam.
o Though the Mughals managed to regain Guwahati briefly after a later
Borphukan deserted it, the Ahoms wrested control in the Battle of Itakhuli in
1682 and maintained it till the end of their rule.
PRELIMS QUESTION
12.Consider the following statements:
1. Mughal Empire reached its maximum territorial extent under the rule of
Aurangazeb.
40 | iLearn IAS
2. Battle of Chausa was fought between Akbar and Rana Pratap.
Which among the above statements is/are correct?
(a) 1 only
(b) 2 only
(c) Both 1 and 2
(d) Neither 1 nor 2
Answer: A

Chang’e-5
Why in news?
o China’s Chang’e-5 lunar mission was launched recently.
What is the Chang’e-5 mission?
o Chang’e-5 probe, is the Chinese National Space Administration’s (CNSA) lunar
sample return mission that is set to launch on November 24 from the
Wenchang Space Launch Center on Hainan Island in China.
o The goal of the mission is to land in the Mons Rumker region of the moon,
where it will operate for one lunar day, which is two weeks long and return a 2
kg sample of the lunar rock possibly by digging about 2 metres deep into the
surface of the Moon.
o If successful it will become the first probe in over four decades to bring back
samples of lunar rock from a previously unexplored portion of the Moon.
o The mission comprises a lunar orbiter, a lander and an ascent probe that will
lift the lunar samples back into orbit and return them back to Earth.
o Chang’e-5 comprises a robotic arm, a coring drill, a sample chamber and is also
equipped with a camera, penetrating radar and a spectrometer.
o The spacecraft is set to return to Earth around December 15.

PRELIMS QUESTION
13.Consider the following statements:
1. Voyager 1 and 2 were spacecrafts launched by NASA to study Jupiter and
41 | iLearn IAS
Saturn.
2. OSIRIS-Rex, a NASA spacecraft was the first to land on an asteroid.
Which among the above statements is/are correct?
(a) 1 only
(b) 2 only
(c) Both 1 and 2
(d) Neither 1 nor 2
Answer: A

Guardian Drones
Why in news?
o The Indian Navy has inducted two MQ-9B Sea Guardian unarmed drones
procured from the U.S. on lease.
About the drones:
o The Guardian is the maritime variant of the Predator MQ-9 Unmanned Aerial
Vehicle (UAV).
o The General Atomics MQ-9 Reaper (sometimes called Predator B) is an
unmanned aerial vehicle (UAV) capable of remotely controlled or autonomous
flight operations.
o It has a maximum endurance of 40 hours and a maximum flying altitude of
40,000 feet.
o It has a 3600 maritime surveillance radar and an optional multimode maritime
surface search radar.
o It was developed by General Atomics Aeronautical Systems (GA-ASI) primarily
for the United States Air Force (USAF). The MQ-9 and other UAVs are referred
to as Remotely Piloted Vehicles/Aircraft (RPV/RPA) by the USAF to indicate
their human ground controllers.

About Drones in Indian Armed forces:


o Till now Indian services have been using the fixed-wing drones just for ISR
(Intelligence, Surveillance, and Reconnaissance) purposes.
42 | iLearn IAS
o IAI Searcher and Heron UAVs from Israel and Guardian from USA are the main
imported drones used.
o DRDO NISHANT, Micro UAVs namely Black Kite, Golden Hawk and Pushpak are
also used.
PRELIMS QUESTION
14.Consider the following pairs of defence platforms and their originating country:
1. Phalcon AWACS : Israel
2. S-400 Triumf : Russia
3. P-8I Aircrafts : France
Which of the pairs given above is/are correct?
(a) 1 and 2 only
(b) 1 and 3 only
(c) 2 and 3 only
(d) 1,2,3 only
Answer: A

Pradhan Mantri Kisan Sampada Yojana (PMKSY)


Why in news?
o Central government recently approved a grant of Rs 107.42 crore for
implementing 28 food processing projects spread over 10 states that are likely
to generate jobs for nearly 10,000 people.
About the scheme:
o It is a Central Sector Scheme – Pradhan mantri kisan sampada yojana
(PMKSY)with an allocation of Rs. 6,000 crores for the period 2016-20.
o It is an umbrella scheme implemented by Ministry of Food Processing
Industries and aims at increasing value addition in agricultural sector.
o The scheme is expected to benefit 20 lakh farmers and generate 5,30,500
direct/ indirect employments by the year 2019-20.
o The PMKSY has the following schemes:
o Mega Food Parks
o Integrated Cold Chain and Value Addition Infrastructure
o Creation / Expansion of Food Processing & Preservation Capacities
o Infrastructure for Agro-processing Clusters
o Creation of Backward and Forward Linkages
o Food Safety and Quality Assurance Infrastructure
o Human Resources and Institutions
o Ministry of Food Processing Industries (MoFPI) had sanctioned 39 Mega Food
Parks and 298 Integrated Cold Chain Projects throughout the country as of
February 2020 under the scheme.

43 | iLearn IAS
Important Sub-schemes:
o Scheme of Cold Chain, Value Addition and Preservation Infrastructure
o It is to provide integrated cold chain and preservation infrastructure
facilities, without any break, from the farm gate to the consumer.
o It covers creation of infrastructure facility along the entire supply chain
viz. pre-cooling, weighing, sorting, grading, waxing facilities at farm
level, multi product/ multi temperature cold storage, CA storage,
packing facility, IQF, blast freezing in the distribution hub and reefer
vans, mobile cooling units for facilitating distribution of horticulture,
organic produce, marine, dairy, meat and poultry et
o Infrastructure for Agro-processing Clusters
o The scheme aims at development of modern infrastructure and
common facilities to encourage group of entrepreneurs to set up food
processing units based on cluster approach by linking groups of
producers/ farmers to the processors and markets through well-
equipped supply chain with modern infrastructure.
o Each agro processing clusters under the scheme have two basic
components i.e. Basic Enabling Infrastructure (roads, water supply,
power supply, drainage, ETP etc.), Core Infrastructure/ Common
facilities (ware houses, cold storages, IQF, tetra pack, sorting, grading
etc) and at least 5 food processing units with a minimum investment of
Rs. 25 crore.
o The units are set up simultaneous along with creation of common
infrastructure.
o At least 10 acres of land is required to be arranged either by purchase or
on lease for at least 50 years for setting up of Agro Processing Cluster.
o Creation of Backward and Forward Linkages
o The objective of the scheme is to provide effective and seamless
backward and forward integration for processed food industry by
plugging the gaps in supply chain in terms of availability of raw material
and linkages with the market.
o Under the scheme, financial assistance is provided for setting up of
primary processing centers/ collection centers at farm gate and
modern retail outlets at the front end along with connectivity through
insulated/ refrigerated transport.
o The scheme is implemented by agencies/ organizations such as Govt./
PSUs/ Joint Ventures/ NGOs/ Cooperatives/ SHGs / FPOs / Private
Sector / individuals etc.
PRELIMS QUESTION
15.Consider the following statements:
1. Codex Alimentarius is food safety standard developed by FSSAI for food
44 | iLearn IAS
processing sector in India.
2. Indian government has an aim to double milk processing in India by 2025.
Which among the above statements is/are correct?
(a) 1 only
(b) 2 only
(c) Both 1 and 2
(d) Neither 1 nor 2
Answer: B

Nagaland Tamarillos
Why in news?
o Nagaland government is promoting the cultivation of Nagaland Tamarillos.
About Tamarillos:
o The tree tomato is locally known as sei bangenuo or khwüdi and the North
Eastern Regional Agricultural Marketing Corp Ltd has been promoting the tree
among farmers.
o Tree tomato (tamarillo) or Solanum betaceum has its origin in South America
but is now found abundantly in kitchen gardens in all districts of the Naga hills,
including Kohima.
o It is a small, evergreen tree and the fruit, in comparison to regular tomatoes is
richer in carbohydrates, protein, minerals and fibre.
o The tree tomato is so popular in Nagaland that the state government got a
certificate for Geographical Indication (GI) for it in 2015.
o Tree tomato in Nagaland meets these requirements because it has a long
history of cultivation and since the Naga tree tomato is usually grown in kitchen
gardens and farmers raise the seedlings from the mature old plants, the purity
of the crop in the state has been maintained.
o Other than the tree tomato, Nagaland also has the GI tag for Naga King Chilli
(raja mircha).
PRELIMS QUESTION
16.Consider the following statements:
1. Geographical indication tags in India are approved by an office under
Department for Promotion of Industry and Internal Trade.
2. GI tags are given in the sectors of handicrafts, agricultural products and
textiles only.
Which among the above statements is/are correct?
(a) 1 only
(b) 2 only
(c) Both 1 and 2
(d) Neither 1 nor 2

45 | iLearn IAS
Answer: A

SDG Investor Map by Invest India


Why in news?
o UNDP and Invest India have launched the SDG Investor Map for India, laying
out 18 Investment Opportunities Areas (IOAs) in six critical SDG enabling
sectors, that can help India push the needle forward on Sustainable
Development.
About Investor Map:
o It lays out 18 Investment Opportunities Areas (IOAs) in six critical SDG
enabling sectors, that can help India push the needle forward on Sustainable
Development.
o Of the 18 IOAs identified, 10 are already mature investable areas that have
seen robust Private Equity and Venture Capital activity, and feature companies
that have been able to unlock scale and demonstrate profitability.
o The remaining eight IOAs are emerging opportunities, which have seen
traction from early-stage investors.
o The map has also identified eight White Spaces, which have seen investor
interest and have the potential to grow into IOAs within a 5-6-year horizon.
However, these require further policy support and private sector participation
to mature into commercially attractive IOAs.
o Notable IOAs include ‘Online Supplementary Education for K12’ (Education),
‘Tech-Enabled Remote Care Services’ (Healthcare), ‘Digital Platforms to service
input/output needs of farmers to enable easy access to markets’ (Agriculture)
and ‘Access to credit by Micro, Small and Medium Enterprises and Low-Income
Groups especially through digital platforms for Income Generating Purposes’
(Financial Services).
About Invest India:
o Invest India is the National Investment Promotion and Facilitation Agency of
India.
o It acts as the first point of reference for investors in India.
o Invest India is set up as a joint venture company between the Department of
Industrial Policy & Promotion (DIPP), Ministry of Commerce & Industry (35%
equity), Federation of Indian Chambers of Commerce and Industry (FICCI)
(51% equity), and State Governments of India (0.5% each).
o Thus, essentially, Invest India is a private company (FICCI owns 51% of shares.)
o It provides sector-specific and state-specific information to a foreign investor,
assists in expediting regulatory approvals, and offers hand-holding services.

46 | iLearn IAS
Objectives of Invest India:
o Transforming the country’s investment climate by simplifying the business
environment for investors.
o Hand-holding investors through their investment lifecycle from pre-investment
to after-care.
o To provide multiple forms of support such as market entry strategies, industry
analysis etc.
PRELIMS QUESTION
17.Consider the following statements regarding India Brand Equity Foundation
(IBEF):
1. India Brand Equity Foundation (IBEF) is an investment promotion agency
under Ministry of Commerce and Industry.
2. While Invest India is a private company, IBEF is fully funded by Union
Government.
Which among the above statements is/are correct?
(a) 1 only
(b) 2 only
(c) Both 1 and 2
(d) Neither 1 nor 2
Answer: C

Antimicrobial resistance (AMR)


Why in news?
o Given the current trajectory, drug resistance/ antimicrobial resistance (AMR)
could lead to 10 million deaths annually, if left unaddressed as per a report by
Wellcome Trust, an independent global charitable foundation.
What is antimicrobial resistance (AMR)?
o Antimicrobial resistance (AMR) is the ability of a microorganism to stop an
antimicrobial (such as antibiotics, antivirals and antimalarials) from working
against it.
o As a result, standard treatments become ineffective, infections persist and
may
spread to others.
o Irrational use of drugs, overdosing or under-dosing, self-medication, misuse of
drugs, and the inappropriate use of antimicrobials etc are the leading reasons
behind
growth of AMR.
o The term antibiotic resistance (AR or ABR) is a subset of AMR, as it applies to
bacteria that become resistant to antibiotics.

47 | iLearn IAS
o Resistant microbes are more difficult to treat, requiring higher doses, or
alternative medications which may prove more toxic.
o These approaches may also be more expensive. Microbes resistant to multiple
antimicrobials are called multidrug resistant (MDR)
Causes:

National level action plan:


o In April 2017, India had released its national level plan on AMR along with
Declaration in Delhi , which called for state-level action plans.
o It was framed by the National Centre for Disease Control and Indian Council of
Medical Research (ICMR).
o The action plan, drawn up with multi-sectoral coordination and in alignment
with the national and global action plans, will be implemented with the
technical support of the World Health Organisation (WHO) and a host of
collaborations with NGOs and pharma majors
About Global AMR R&D Hub:
o The Global Antimicrobial Resistance Research and Development Hub – or
Global AMR R&D Hub for short – is designed to tackle growing Antimicrobial
Resistance (AMR) across the world.
o It was created on the initiative of G-20 leaders.
o It aims to improve the coordination of international efforts and initiatives to
tackle AMR.
o It will also aim to further increasing investments into R&D for AMR.
o The Secretariat of the hub is based in Berlin and is funded by German Federal
Ministry of Education and Research.
48 | iLearn IAS
PRELIMS QUESTION
18.Consider the following statements:
1. MDR-TB is form of Tuberculosis resistant to first-line anti-TB drugs.
2. Longitude Prize is an inducement prize offered for research into solving the
global problem of antimicrobial resistance.
Which among the above statements is/are correct?
(a) 1 only
(b) 2 only
(c) Both 1 and 2
(d) Neither 1 nor 2
Answer: C

National Command, Control, Communication and


Intelligence Centre (NC3I)
Why in news?
o The Navy’s Information Management and Analysis Centre (IMAC), the nodal
agency for maritime data fusion set up after the 26/11 Mumbai terror attacks,
will soon become a National Maritime Domain Awareness (NDMA) centre
o It will thus become a multi-agency centre with all representatives having role in
maritime security having a representation.
About NC3I network:
o National Command, Control, Communication and Intelligence Centre (NC3I) is a
maritime security project.
o It was launched soon after the 26/11 attacks in Mumbai, when Pakistani
terrorists managed to reach the city undetected due to gaps in coastal
surveillance.
o It enables round-the-clock surveillance of Indian coastline by radars, high-
definition electro optic cameras, maritime surveillance aircraft and satellites.
o It connects 20 naval and 31 coast guard stations along the coast.
o It also gets data from Automatic Identification System (AIS) installed on fishing
vessels as also the World Shipping Register which has data of foreign or indian
vessels passing through Indian waters.
o Under the system 46 coastal radar stations at strategic locations in all the nine
littoral states continuously monitor sea lanes.
o These stations in turn relay information to six core centres located in
Gandhinagar (Gujarat), Mumbai, Chennai, Port Blair, Visakhapatnam and Kochi.
o Final information is sent to Information Management and Analysis Centre
(IMAC) of the navy set up at Gurugram for analysis.

49 | iLearn IAS
IMAC:
o Information Management and Analysis Centre (IMAC) is the Command Centre
or nodal centre of the NC3I network.
o Approved by the Defence Acquisition Council in 2012, the IMAC became
operational in 2014 at a cost of ₹450 crore and is located in Gurugram.
o An officer sitting at his console in the Gurgaon centre can access real time
information on his computer screen about traffic of ships and boats in his
designated zone, be it waters of Chennai or remote islands in Andaman &
Nicobar
o It uses a customised, state of the art ‘Decision making or Decision support
software’ which enables analysing various inputs and disseminating them
among concerned agencies to ensure a gapless surveillance of the entire
coastline.
PRELIMS QUESTION
19.Consider the following statements:
1. Indian coastline is more than 6000 kilometre long.
2. Indian coast guard operates under Ministry of defence.
Which among the above statements is/are correct?
(a) 1 only
(b) 2 only
(c) Both 1 and 2
(d) Neither 1 nor 2
Answer: C

Sangai deer
Why in News?
o Sangai festival, an annual cultural festival organised by Manipur Tourism
Department has been cancelled due to the pandemic.
Sangai deer
o Sangai (the brow-antlered deer) is an endemic and endangered subspecies of
brow-antlered deer found only in Manipur (Keibul Lamjao National Park)
o It is largely seen over the floating biomass, locally called “phumdi” in the South
Eastern part of Loktak Lake inside the park.
o They have uniquely distinctive antlers,
measuring 100–110 cm. in length with
extremely long brow tine.
o It is also the state animal of Manipur.
o Culturally, the Sangai finds itself imbedded
deep into the legends and folklore of the
Manipuris.
50 | iLearn IAS
o Sangai is interpreted as the binding soul between humans and the
nature.
Conservation Issues
o Sangai deer is classified as Endangered on IUCN Red List.
o Degenerating habitat of phumdi as a result of
o Continuous inundation and flooding caused due to artificial reservoir.
o Water quality of the reservoir is degrading due to pollution and
stoppage of nutrient supply.
o There is also invasion of non-native plants like Paragrass.
o Sangai also faces threats of diseases from the livestock, inbreeding depression
and poaching.
PRELIMS QUESTION
20.Consider the following statements:
1. Keibul Lamjao National Park is the smallest national park in India, by area.
2. Chiru, a type of antelope is seen extensively in Keibul Lamjao National Park.
Which among the above statements is/are correct?
(a) 1 only
(b) 2 only
(c) Both 1 and 2
(d) Neither 1 nor 2
Answer: D

Dooars
Why in news?
o Instances of Human-animal conflict in Dooars region is likely to increase as the
area under cultivation has increased.
o The increase in cultivation is primarily due to the return of Labourers into West
Bengal and Assam from states like Kerala due to the pandemic.
About Dooars:
o The Dooars or Duars are the alluvial floodplains in eastern-northeastern India
that lie south of the outer foothills of the Himalayas and north of the
Brahmaputra River basin.
o This region is about 30 km wide and stretches over about 350 km from the
Teesta River in West Bengal to the Dhansiri River in Assam.
o The region forms the gateway to Bhutan and is part of the Terai-Duar savanna
and grasslands ecoregion.
o Dooars means 'doors' in Assamese, Bengali, Maithili, Bhojpuri, and Magahi
languages.
o There are 18 such passages between the hills in Bhutan and the plains in India.

51 | iLearn IAS
o This region is divided by the Sankosh River into Eastern and Western Dooars.
also known as the Assam Dooars, and the Bengal Dooars respectively.
o The economy of Dooars is based on three "T"s – Tea, Tourism and Timber and
the main industry of the Dooars region is the tea.
o Many wildlife sanctuaries and national park like Manas National Park in Assam,
Jaldapara National Park, Buxa Tiger Reserve, Gorumara National Park,
Chapramari Wildlife Sanctuary, Singalila National Park, Neora Valley National
Park, and the Mahananda Wildlife Sanctuary in West Bengal are located in this
region.
PRELIMS QUESTION
21.Consider the following statements:
1. India is the largest producer of tea in the world.
2. Darjeeling tea was India's first product to receive geographical indication
protection.
Which among the above statements is/are correct?
(a) 1 only
(b) 2 only
(c) Both 1 and 2
(d) Neither 1 nor 2
Answer: B

Blue Tides
Why in news?
o Blue Tide has been Spotted Along Juhu, Ratnagiri Beaches.
About Blue Tides:
o The phenomenon called 'blue tide' happens when luminescent marine life
makes the sea appear a deep shade of blue.
o The spectacle occurs when phytoplankton (microscopic marine plants),
commonly known as dinoflagellates, produce light through chemical reactions
in proteins.
o Waves disturb these unicellular microorganisms and makes them release blue
light.
o Although beautiful the tides may also be a signal of danger as many of the
species in this group are toxic.
o If dinoflagellates reproduce rapidly, they may cause so-called ‘red tides’ which
are associated with wildlife mortalities and harmful human exposure.
o The production of natural toxins such as brevetoxins and ichthyotoxins are
harmful to marine life.

52 | iLearn IAS
o The primary cause of these blooms is eutrophication which ocean makes
overly enriched with minerals and nutrients which induce excessive growth of
algae.
What is Bioluminescence?
o Bioluminescence is the property of a living organism to produce and emit light.
o Animals, plants, fungi, bacteria and a remarkable diversity of marine animals
and microbes show bioluminescence. are able to produce their own light.
o It is found in many marine organisms such as bacteria, algae, jellyfish, worms,
crustaceans, sea stars, fish and sharks. Luminescence is generally higher in
deep-living and planktonic organisms than in shallow species.
PRELIMS QUESTION
22.Consider the following statements:
1. Oligotrophic lakes host a larger number of organisms when compared to
eutrophic lakes.
2. Mesotrophic lakes are lakes with an intermediate level of productivity.
Which among the above statements is/are correct?
(a) 1 only
(b) 2 only
(c) Both 1 and 2
(d) Neither 1 nor 2
Answer: B

Brain Electrical Oscillation Signature Profiling (BEOSP)


Why in news?
o The accused in Hathras rape case in Uttar Pradesh will undergo brain
fingerprinting.
What is BEOSP test?
o Brain Electrical Oscillation Signature Profiling (BEOSP) also known as brain
fingerprinting is a neuro psychological method of interrogation in which the
accused’s participation in the crime is investigated by studying their brain’s
response.
o The BEOSP test is carried out via a process known as electroencephalogram,
conducted to study the electrical behaviour of the human brain.
o Under this test, the consent of the accused is first taken and they are then
made to wear caps with dozens of electrodes attached to them.
o The accused are then shown visuals or played audio clips related to the crime
to check if there is any triggering of neurons in their brains which then generate
brainwaves.
o The test results are then studied to determine the participation of the accused
in a crime.

53 | iLearn IAS
BEOSP test vs polygraph?
o The BEOSP procedure does not involve a question answer session with the
accused and is rather a neuro psychological study of their brain.
o In a polygraph test, the accused person’s physiological indicators are taken into
account which include blood pressure, pulse rate, respiration and skin
conductivity.
o However experts say that while a person might be able to control their pulse
rate and BP even in times of distress, a BEOSP test offers a much more credible
result.
Can these tests be admitted as evidence?
o Not as a standalone.
o In 2010, the Supreme Court passed a judgment in the Selvi versus State of
Karnataka case where the bench observed that narco analysis, polygraph and
brain mapping tests cannot be forced upon any individual without their consent
and the test results cannot be admitted solely as evidence.
o However, any information or material discovered during the tests can be
made part of the evidence.
PRELIMS QUESTION
23.National Forensic Sciences University is located in:
(a) Rajasthan
(b) Gujarat
(c) Haryana
(d) Uttar Pradesh
Answer: B

Farmers Producer Organisations


Why in news?
o Honey FPO Programme of National Agricultural Cooperative Marketing
Federation of India Limited. (NAFED) was inaugurated by Minister of
Agriculture and Farmers’ Welfare
o Through these Honey FPOs, NAFED will work for promotion of beekeeping as
an occupation for unemployed women and tribal populations and uplift their
livelihood.
o It will also act as an intermediary and filling up the gaps between the elements
of the beekeeping supply chain and also ensure price remuneration to the
beekeeping farmers.
What are FPOs?
o A Producer Organisation (PO) is a legal entity formed by primary producers,
viz. farmers, milk producers, fishermen, weavers, rural artisans, craftsmen.

54 | iLearn IAS
o A PO can be a producer company, a cooperative society, a Trust, a Society or
any other legal form which provides for sharing of profits/benefits among the
members.
o Farmers Producer Organisation is simply a PO of farmers.
o Small Farmers’ Agribusiness Consortium (SFAC) is providing support for
promotion of FPOs.
What are the essential features of an FPO?
o It is formed by a group of producers for either farm activities.
o It is a registered body and a legal entity.
o Producers are shareholders and thus owners of the organization.
o It deals with business activities related to the primary produce/product.
o It works for the benefit of the member producers.
o A part of the profit is shared amongst the producers.
o Rest of the surplus is added to its owned funds for business expansion.
About SFAC:
o The Government established Small Farmers’ Agri-Business Consortium (SFAC)
as a Society in 1994.
o It is an Autonomous Society promoted by Ministry of Agriculture, Cooperation
and Farmers’ Welfare.
o The Society is governed by Board of Management, which is chaired, ex-officio,
by Union Minister for Agriculture and Farmers Welfare as the President.
o It aims to facilitate agri-business ventures by catalysing private investment
through Venture Capital Assistance (VCA) Scheme in close association with
financial institutions.
o It is also the lead implementation agency for National Agriculture Market (e-
NAM) scheme.
About NAFED:
o National Agricultural Cooperative Marketing Federation of India Ltd (NAFED) is
an apex organization of marketing cooperatives for agricultural produce in
India.
o It was founded on the birthday of Mahatma Gandhi on 2 October 1958 to
promote the trade of agricultural produce and forest resources across the
nation.
o It is registered under Multi State Co-operative Societies Act.
o NAFED is now one of the largest procurements as well as marketing agencies
for agricultural products in India.
o NAFED is the nodal agency to implement price stabilization measures under
"Operation Greens" which aims to stabilise the supply of tomato, onion and
potato crops (TOP crops) in India

55 | iLearn IAS
PRELIMS QUESTION
24.Consider the following statements regarding agricultural marketing in India:
1. e-NAM aims to create a national agricultural market by linking APMCs
across India.
2. NAFED is the lead agency for managing Agri Market Infrastructure Fund
(AMIF) created by the central government.
Which among the above statements is/are correct?
(a) 1 only
(b) 2 only
(c) Both 1 and 2
(d) Neither 1 nor 2
Answer: A

Climate Change Knowledge Portal


Why in news?
o Ministry of Environment, Forest and Climate Change, Shri Prakash Javadekar
launched the “India Climate Change Knowledge Portal”.
About the portal:
o The portal captures sector-wise adaptation and mitigation actions that are
being taken by the various line Ministries in one place including updated
information on their implementation.
o The knowledge portal will help in disseminating knowledge among citizens
about all the major steps Government is taking at both national and
international levels to address climate change issues.
o The eight major components included in the knowledge portal are:
o India’s Climate Profile
o National Policy Framework
o India’s NDC goals
o Adaptation Actions
o Mitigation Actions
o Bilateral and Multilateral Cooperation
o International Climate Negotiations
o Reports & Publications
World Bank Climate change knowledge portal:
o This is a portal which provides information on Global climate change actions.
o Climate Change Knowledge Portal (CCKP) provides an online platform for access
to comprehensive global, regional, and country data related to climate change
and development.

56 | iLearn IAS
o It enables users including researchers and policy makers to evaluate climate-
related vulnerabilities, risks, and actions for a particular location on the globe
by interpreting climate and climate-related data at different levels of details.
PRELIMS QUESTION
25.Consider the following statements regarding agricultural marketing in India:
1. Share of renewable energy in India’s electricity production is more than
33%.
2. India has set a target of achieving 40% power production from renewable
energy by 2030.
Which among the above statements is/are correct?
(a) 1 only
(b) 2 only
(c) Both 1 and 2
(d) Neither 1 nor 2
Answer: C

Neom City
Why in news?
o Israeli Prime Minister Benjamin Netanyahu met Saudi Arabia’s Crown Prince
Mohammed bin Salman last week in a meeting in Neom.
About the city:
o Neom is a planned cross-border city in the Tabuk Province of north western
Saudi Arabia.
o It is planned to incorporate smart city technologies and also function as a
tourist destination.
o The site is near the Red Sea, and Strait of Tiran.
o It will cover a total area of 26,500 km2 and will extend 460 km along the coast
of the Red Sea.
o In 2019, Saudi Arabia floated a company named Neom to develop the city’s
economic zone.
o The company is wholly owned by the Public Investment Fund, Saudi Arabia’s
sovereign wealth fund with an estimated asset of $383 billion.
o As part of the Vision 2030 plan, which seeks to diversify the Kingdom’s
economy and reduce its dependency on oil, the $500-billion ambitious project
plans to build a carbon-neutral, technologically advanced city state that can
house up to 1 million people.

57 | iLearn IAS
PRELIMS QUESTION
26.Neom, a planned futuristic city is being constructed in which county?
(a) Saudi Arabia
(b) United Arab Emirates
(c) Turkey
(d) Singapore
Answer: A

Cosmos Malabaricus
Why in news?
o The Indian embassy in the Netherlands, in a tweet, said initiatives are being
taken to discuss the Cosmos Malabaricus project with various stakeholders.
About Cosmos Malabaricus:
o It is a proposed project highlighting Kerala’s history based on 17th-century
Dutch records.
o The Cosmos Malabaricus project is currently under consideration of authorities
from India and the Netherlands.
o It is aimed at making records of already digitised archival material from the
government in the Netherlands accessible to a larger base of readers, including
scholars from India.
o It will be a 7-year collaboration under which archival material pertaining to
Dutch East India Company will be translated and summaries published in
English.
o It will have descriptions of political and military organisations, dynastic
developments, economic matters, social and religious aspects of Kerala
between 1643 and 1852.

58 | iLearn IAS
PRELIMS QUESTION
27.Consider the following statements regarding Dutch East India Company:
1. It was the second European company to establish factories in India, after
the Portuguese.
2. Battle of Colachel lead to decline of Dutch strength on the western coast of
India.
Which among the above statements is/are correct?
(a) 1 only
(b) 2 only
(c) Both 1 and 2
(d) Neither 1 nor 2
Answer: B

Natural Gas sector in India


Why in news?
o Various infrastructure projects related to improvement of utilisation of natural
gas in India has been in news.
Why Natural Gas?
o Natural gas is a superior fuel as compared with coal and other liquid fuels
because it is
o Environment friendly and safer: Natural gas emits less carbon as
compared to coal and other fossil fuels.
o Cheaper fuel: Natural Gas (as CNG) is cheaper by 60% as compared with
petrol. Similarly, Natural Gas (as PNG) is cheaper by 40 % as compared
with market price LPG.
o Save space: Natural Gas is supplied through pipelines just like one gets
water from the tap. There is no need to store cylinders in the kitchen
and thus saves space
Natural Gas scenario in India:
o In FY 2018-19, the total gas consumption in India was around 148. million
standard cubic meters per day (mmscmd).
o The share of domestic gas and imported RLNG was about 48% & 52%
respectively.
o Fertiliser industry is the biggest user of natural gas accounting for 27% of
consumption followed by power (22%) and City gas distribution (17%).
o At present, there are about 17000 km long Natural Gas pipeline network
which is operational in the country.
o In order to make available natural gas across the country, it has been envisaged
to develop additional about 15,000 km pipelines to complete the National Gas
Grid and same are at various stages of development.
59 | iLearn IAS
Gas supply sources in India:
o Domestic Gas Sources:
o The domestic gas in the country is being supplied from the oil & gas
fields located at western and southeastern areas viz. Hazira basin,
Mumbai offshore & KG basin as well as North East Region (Assam &
Tripura).
o Pricing and utilization of domestic gas is as per policies issued by the
Government from time to time.
o In FY 2018-19, total domestic gas production was about 90.05
MMSCMD.
o Import of Liquefied Natural Gas (LNG):
o In order to meet the gas demand, Liquefied Natural Gas (LNG) is
imported through Open General License (OGL) in the country.
o The price and utilization of imported LNG is mutually decided by
buyers and sellers. At present, country is having six (6) operational LNG
regasification terminals operational with capacity of about 38.8 MMTPA
(~ 140 MMSCMD).
About CGD Network
o Government aims to promote the usage of environment friendly clean fuel i.e.
natural gas as a fuel/feedstock across the country to move towards a gas-
based economy.
o Accordingly, development of CGD networks has been focused to increase the
availability of cleaner cooking fuel (i.e. PNG) and transportation fuel (i.e. CNG)
to the citizens of the country.
o The expansion of CGD network will also benefit to industrial and commercial
units by ensuring the uninterrupted supply of natural gas.
What is Compressed Natural Gas (CNG)?
o CNG is made by compressing natural gas, which is mainly composed of
methane (CH4), to less than 1% of the volume it occupies at standard
atmospheric pressure.
o It is a fuel that can be used in place of gasoline, diesel fuel and liquefied
petroleum gas (LPG).
o CNG combustion produces fewer undesirable gases than the aforementioned
fuels.
o In comparison to other fuels, natural gas poses less of a threat in the event of
a spill, because it is lighter than air and disperses quickly when released.
What is Liquefied natural gas (LNG)?
o Liquefied natural gas (LNG) is natural gas that has been cooled down to liquid
form for ease and safety of non-pressurized storage or transport.
o It takes up about 1/600th the volume of natural gas in the gaseous state
60 | iLearn IAS
o It is odorless, colorless, non-toxic and non-corrosive.
o Natural gas is mainly converted to LNG for transport over the seas where
laying pipelines is not feasible technically and economically.
o Specially designed cryogenic sea vessels (LNG carriers) or cryogenic road
tankers are used for LNG transport.
o LNG is principally used for transporting natural gas to markets, where it is
regasified and distributed as pipeline natural gas.

PRELIMS QUESTION
28.Consider the following statements:
1. Natural Gas is a cleaner source of energy when compared to petroleum.
2. More than 40% of natural gas demand in India is being met from domestic
sources.
Which among the above statements is/are correct?
(a) 1 only
(b) 2 only
(c) Both 1 and 2
(d) Neither 1 nor 2
Answer: C

61 | iLearn IAS

You might also like